How Does Green's Theorem Simplify Calculating a Line Integral for an Ellipse?

Click For Summary
Green's Theorem can be applied to evaluate the line integral along the positively oriented curve of the ellipse defined by x² + 2y² = 2. To simplify the calculation, one can consider a change of variables to transform the ellipse into a circle, although this approach may introduce unnecessary complexity due to the need to calculate the Jacobian. Instead, it is recommended to directly parametrize the ellipse and determine the limits for the integral without additional transformations. Understanding the relationship between the parameters and the ellipse's geometry can streamline the evaluation process. Ultimately, leveraging Green's Theorem effectively simplifies the computation of the line integral.
Calpalned
Messages
297
Reaction score
6

Homework Statement


Use Green's Theorem to evaluate the line integral along the given positively oriented curve. ##\int_C y^4 dx + 2xy^3 dy ##, C is the ellipse ##x^2 + 2y^2 = 2##.

Homework Equations


Change of variables: ##\int \int_S f(x(u,v),y(u,v)) |{\frac {\partial(x,y)}{\partial (u,v)}}| du dv ##

The Attempt at a Solution


How do I change the ellipse to a circle? Is there a way to determine u and v?
 
Physics news on Phys.org
Please refer back to post #3 in your prior thread:

https://www.physicsforums.com/threads/using-greens-theorem.810989/

You could have posted there.

Anyway, after parametrizing the ellipse, you should know what the limits on the integral are for ##r## and ##\theta## without very much thought.

While a change of variables ##x = au## and ##y = bv## would map the ellipse to a circle of radius ##\sqrt{2}##, this is unnecessary and a bit of extra work since you still need to calculate the Jacobian.

I suggest just thinking about it for a second.
 
Question: A clock's minute hand has length 4 and its hour hand has length 3. What is the distance between the tips at the moment when it is increasing most rapidly?(Putnam Exam Question) Answer: Making assumption that both the hands moves at constant angular velocities, the answer is ## \sqrt{7} .## But don't you think this assumption is somewhat doubtful and wrong?

Similar threads

Replies
3
Views
2K
  • · Replies 10 ·
Replies
10
Views
2K
Replies
12
Views
2K
  • · Replies 1 ·
Replies
1
Views
2K
  • · Replies 8 ·
Replies
8
Views
2K
  • · Replies 4 ·
Replies
4
Views
2K
  • · Replies 2 ·
Replies
2
Views
1K
  • · Replies 7 ·
Replies
7
Views
2K
  • · Replies 4 ·
Replies
4
Views
3K
  • · Replies 15 ·
Replies
15
Views
2K